Can the symmetric group $S_n$ be imbedded as a subgroup in $A_n+1$? Announcing the arrival of Valued Associate #679: Cesar Manara Planned maintenance scheduled April 17/18, 2019 at 00:00UTC (8:00pm US/Eastern)$A_n$ contains an isomorphic copy of $S_n-2$No subgroups of $A_n+1$ is isomorphic to $S_n$Show that for $nge 2$, $S_n$ can not be imbedded in to $A_n+1$Rotman's exercise 2.8 “$S_n$ cannot be imbedded in $A_n+1$”Does group $A_6$ contain subgroup isomorphic with $S_4$Embedding $S_n$ in $A_2n$Show that $S_n$ is never isomorphic to a subgroup of $A_n+1$ for $nge 2$$S_n$ embeds into $A_n+2$.Some questions on the Frattini subgroupWhat is the group generated by the conjugacy class containing $(12ldots n)$ in $S_n$?Show group of order $4n + 2$ has a subgroup of index 2.Rotman's exercise 2.8 “$S_n$ cannot be imbedded in $A_n+1$”Generating the symmetric group $S_n$Can a group have a subset that is stable under all automorphisms, but not under inverse?Covering finite groups by unions of proper subgroupsWhat's the group with 2 elements: one for even permutations and one for odd permutations (on $n$ points)? How to construct it?Show that $S_n$ is never isomorphic to a subgroup of $A_n+1$ for $nge 2$Show that for $nge 2$, $S_n$ can not be imbedded in to $A_n+1$

If a contract sometimes uses the wrong name, is it still valid?

3 doors, three guards, one stone

Withdrew £2800, but only £2000 shows as withdrawn on online banking; what are my obligations?

How can I make names more distinctive without making them longer?

Identify plant with long narrow paired leaves and reddish stems

What LEGO pieces have "real-world" functionality?

Can an alien society believe that their star system is the universe?

Error "illegal generic type for instanceof" when using local classes

A coin, having probability p of landing heads and probability of q=(1-p) of landing on heads.

What to do with chalk when deepwater soloing?

What is the meaning of the new sigil in Game of Thrones Season 8 intro?

How do I stop a creek from eroding my steep embankment?

How to find out what spells would be useless to a blind NPC spellcaster?

Should I use a zero-interest credit card for a large one-time purchase?

How do I keep my slimes from escaping their pens?

What is the role of the transistor and diode in a soft start circuit?

Bete Noir -- no dairy

Is the Standard Deduction better than Itemized when both are the same amount?

What is a non-alternating simple group with big order, but relatively few conjugacy classes?

How does debian/ubuntu knows a package has a updated version

How to bypass password on Windows XP account?

Can I cast Passwall to drop an enemy into a 20-foot pit?

Generate an RGB colour grid

How to tell that you are a giant?



Can the symmetric group $S_n$ be imbedded as a subgroup in $A_n+1$?



Announcing the arrival of Valued Associate #679: Cesar Manara
Planned maintenance scheduled April 17/18, 2019 at 00:00UTC (8:00pm US/Eastern)$A_n$ contains an isomorphic copy of $S_n-2$No subgroups of $A_n+1$ is isomorphic to $S_n$Show that for $nge 2$, $S_n$ can not be imbedded in to $A_n+1$Rotman's exercise 2.8 “$S_n$ cannot be imbedded in $A_n+1$”Does group $A_6$ contain subgroup isomorphic with $S_4$Embedding $S_n$ in $A_2n$Show that $S_n$ is never isomorphic to a subgroup of $A_n+1$ for $nge 2$$S_n$ embeds into $A_n+2$.Some questions on the Frattini subgroupWhat is the group generated by the conjugacy class containing $(12ldots n)$ in $S_n$?Show group of order $4n + 2$ has a subgroup of index 2.Rotman's exercise 2.8 “$S_n$ cannot be imbedded in $A_n+1$”Generating the symmetric group $S_n$Can a group have a subset that is stable under all automorphisms, but not under inverse?Covering finite groups by unions of proper subgroupsWhat's the group with 2 elements: one for even permutations and one for odd permutations (on $n$ points)? How to construct it?Show that $S_n$ is never isomorphic to a subgroup of $A_n+1$ for $nge 2$Show that for $nge 2$, $S_n$ can not be imbedded in to $A_n+1$










8












$begingroup$


I'm working through Rotman (1994). It's the 2nd chapter where my curiosity lead me to a question that is not from the book. Here's the problem that inspired the question:




2.8. Imbed $S_n$ as a subgroup of $A_n+2$, but show, for $nge2$, that $S_n$ cannot be imbedded in $A_n+1.$




Since the alternate subgroup has the property of even parity, I thought that a little investigation of parity for the second and third symmetric groups was in order. So I constructed a table for each of them, grouped by even and odd number of disjoint products.



I'll forego writing the actual table and simply provide the answers. In symmetric groups with an even idegree, two even permutations come together under composition to produce an even permutation. Two odds also make an even. Two different parities produce odd elements.



When the symmetric group has an odd degree, e.g. $S_3$, two odds make an odd, two evens make an odd, and two different parities make an even element. Exactly the opposite results from the symmetric groups with even index.



Two questions: is this enough to prove the $A_n+1$ case? And can this be generalized modulo 2 e.g. $A_2n+1$?










share|cite|improve this question











$endgroup$











  • $begingroup$
    You aren't computing the parity correctly. The multiplication rule should be the same for all $S_n$. Also, $A_n$ embeds into $A_n+1$ for all $n$.
    $endgroup$
    – Qiaochu Yuan
    Nov 17 '11 at 19:48











  • $begingroup$
    @bwkaplan: Why did you change back the title? The question asks that you embed $S_n$ into $A_n+2$, not into $A_n+3$. Your entire question never mentions $A_n+3$.
    $endgroup$
    – Arturo Magidin
    Nov 17 '11 at 20:04










  • $begingroup$
    @ArturoMagidin, I am more interested in the $A_2n+1$ groups, with the first one e.g. $A_n+1$ being a special case. See the last paragraph. Anyway, I'm going off to recalculate the parity again and will be back if I have any more questions. thanks again!
    $endgroup$
    – user4536
    Nov 17 '11 at 20:18










  • $begingroup$
    @bwkaplan: I think only Qiaochu Yuan has read your (mind to be able to read your) actual question, but perhaps his answer was not direct enough. Yes, Sn is isomorphic to a subgroup of A(n+3). You answer the exercise to get Sn isomorphic to a subgroup of A(n+2), and then use QY's answer that A(n+2) is isomorphic to a subgroup of A(n+3).
    $endgroup$
    – Jack Schmidt
    Nov 17 '11 at 20:22











  • $begingroup$
    @bwkaplan: Still, $2n+1neq n+3$...
    $endgroup$
    – Arturo Magidin
    Nov 17 '11 at 20:30















8












$begingroup$


I'm working through Rotman (1994). It's the 2nd chapter where my curiosity lead me to a question that is not from the book. Here's the problem that inspired the question:




2.8. Imbed $S_n$ as a subgroup of $A_n+2$, but show, for $nge2$, that $S_n$ cannot be imbedded in $A_n+1.$




Since the alternate subgroup has the property of even parity, I thought that a little investigation of parity for the second and third symmetric groups was in order. So I constructed a table for each of them, grouped by even and odd number of disjoint products.



I'll forego writing the actual table and simply provide the answers. In symmetric groups with an even idegree, two even permutations come together under composition to produce an even permutation. Two odds also make an even. Two different parities produce odd elements.



When the symmetric group has an odd degree, e.g. $S_3$, two odds make an odd, two evens make an odd, and two different parities make an even element. Exactly the opposite results from the symmetric groups with even index.



Two questions: is this enough to prove the $A_n+1$ case? And can this be generalized modulo 2 e.g. $A_2n+1$?










share|cite|improve this question











$endgroup$











  • $begingroup$
    You aren't computing the parity correctly. The multiplication rule should be the same for all $S_n$. Also, $A_n$ embeds into $A_n+1$ for all $n$.
    $endgroup$
    – Qiaochu Yuan
    Nov 17 '11 at 19:48











  • $begingroup$
    @bwkaplan: Why did you change back the title? The question asks that you embed $S_n$ into $A_n+2$, not into $A_n+3$. Your entire question never mentions $A_n+3$.
    $endgroup$
    – Arturo Magidin
    Nov 17 '11 at 20:04










  • $begingroup$
    @ArturoMagidin, I am more interested in the $A_2n+1$ groups, with the first one e.g. $A_n+1$ being a special case. See the last paragraph. Anyway, I'm going off to recalculate the parity again and will be back if I have any more questions. thanks again!
    $endgroup$
    – user4536
    Nov 17 '11 at 20:18










  • $begingroup$
    @bwkaplan: I think only Qiaochu Yuan has read your (mind to be able to read your) actual question, but perhaps his answer was not direct enough. Yes, Sn is isomorphic to a subgroup of A(n+3). You answer the exercise to get Sn isomorphic to a subgroup of A(n+2), and then use QY's answer that A(n+2) is isomorphic to a subgroup of A(n+3).
    $endgroup$
    – Jack Schmidt
    Nov 17 '11 at 20:22











  • $begingroup$
    @bwkaplan: Still, $2n+1neq n+3$...
    $endgroup$
    – Arturo Magidin
    Nov 17 '11 at 20:30













8












8








8


6



$begingroup$


I'm working through Rotman (1994). It's the 2nd chapter where my curiosity lead me to a question that is not from the book. Here's the problem that inspired the question:




2.8. Imbed $S_n$ as a subgroup of $A_n+2$, but show, for $nge2$, that $S_n$ cannot be imbedded in $A_n+1.$




Since the alternate subgroup has the property of even parity, I thought that a little investigation of parity for the second and third symmetric groups was in order. So I constructed a table for each of them, grouped by even and odd number of disjoint products.



I'll forego writing the actual table and simply provide the answers. In symmetric groups with an even idegree, two even permutations come together under composition to produce an even permutation. Two odds also make an even. Two different parities produce odd elements.



When the symmetric group has an odd degree, e.g. $S_3$, two odds make an odd, two evens make an odd, and two different parities make an even element. Exactly the opposite results from the symmetric groups with even index.



Two questions: is this enough to prove the $A_n+1$ case? And can this be generalized modulo 2 e.g. $A_2n+1$?










share|cite|improve this question











$endgroup$




I'm working through Rotman (1994). It's the 2nd chapter where my curiosity lead me to a question that is not from the book. Here's the problem that inspired the question:




2.8. Imbed $S_n$ as a subgroup of $A_n+2$, but show, for $nge2$, that $S_n$ cannot be imbedded in $A_n+1.$




Since the alternate subgroup has the property of even parity, I thought that a little investigation of parity for the second and third symmetric groups was in order. So I constructed a table for each of them, grouped by even and odd number of disjoint products.



I'll forego writing the actual table and simply provide the answers. In symmetric groups with an even idegree, two even permutations come together under composition to produce an even permutation. Two odds also make an even. Two different parities produce odd elements.



When the symmetric group has an odd degree, e.g. $S_3$, two odds make an odd, two evens make an odd, and two different parities make an even element. Exactly the opposite results from the symmetric groups with even index.



Two questions: is this enough to prove the $A_n+1$ case? And can this be generalized modulo 2 e.g. $A_2n+1$?







group-theory






share|cite|improve this question















share|cite|improve this question













share|cite|improve this question




share|cite|improve this question








edited Feb 18 '16 at 21:40









Dietrich Burde

82.1k649107




82.1k649107










asked Nov 17 '11 at 19:42







user4536


















  • $begingroup$
    You aren't computing the parity correctly. The multiplication rule should be the same for all $S_n$. Also, $A_n$ embeds into $A_n+1$ for all $n$.
    $endgroup$
    – Qiaochu Yuan
    Nov 17 '11 at 19:48











  • $begingroup$
    @bwkaplan: Why did you change back the title? The question asks that you embed $S_n$ into $A_n+2$, not into $A_n+3$. Your entire question never mentions $A_n+3$.
    $endgroup$
    – Arturo Magidin
    Nov 17 '11 at 20:04










  • $begingroup$
    @ArturoMagidin, I am more interested in the $A_2n+1$ groups, with the first one e.g. $A_n+1$ being a special case. See the last paragraph. Anyway, I'm going off to recalculate the parity again and will be back if I have any more questions. thanks again!
    $endgroup$
    – user4536
    Nov 17 '11 at 20:18










  • $begingroup$
    @bwkaplan: I think only Qiaochu Yuan has read your (mind to be able to read your) actual question, but perhaps his answer was not direct enough. Yes, Sn is isomorphic to a subgroup of A(n+3). You answer the exercise to get Sn isomorphic to a subgroup of A(n+2), and then use QY's answer that A(n+2) is isomorphic to a subgroup of A(n+3).
    $endgroup$
    – Jack Schmidt
    Nov 17 '11 at 20:22











  • $begingroup$
    @bwkaplan: Still, $2n+1neq n+3$...
    $endgroup$
    – Arturo Magidin
    Nov 17 '11 at 20:30
















  • $begingroup$
    You aren't computing the parity correctly. The multiplication rule should be the same for all $S_n$. Also, $A_n$ embeds into $A_n+1$ for all $n$.
    $endgroup$
    – Qiaochu Yuan
    Nov 17 '11 at 19:48











  • $begingroup$
    @bwkaplan: Why did you change back the title? The question asks that you embed $S_n$ into $A_n+2$, not into $A_n+3$. Your entire question never mentions $A_n+3$.
    $endgroup$
    – Arturo Magidin
    Nov 17 '11 at 20:04










  • $begingroup$
    @ArturoMagidin, I am more interested in the $A_2n+1$ groups, with the first one e.g. $A_n+1$ being a special case. See the last paragraph. Anyway, I'm going off to recalculate the parity again and will be back if I have any more questions. thanks again!
    $endgroup$
    – user4536
    Nov 17 '11 at 20:18










  • $begingroup$
    @bwkaplan: I think only Qiaochu Yuan has read your (mind to be able to read your) actual question, but perhaps his answer was not direct enough. Yes, Sn is isomorphic to a subgroup of A(n+3). You answer the exercise to get Sn isomorphic to a subgroup of A(n+2), and then use QY's answer that A(n+2) is isomorphic to a subgroup of A(n+3).
    $endgroup$
    – Jack Schmidt
    Nov 17 '11 at 20:22











  • $begingroup$
    @bwkaplan: Still, $2n+1neq n+3$...
    $endgroup$
    – Arturo Magidin
    Nov 17 '11 at 20:30















$begingroup$
You aren't computing the parity correctly. The multiplication rule should be the same for all $S_n$. Also, $A_n$ embeds into $A_n+1$ for all $n$.
$endgroup$
– Qiaochu Yuan
Nov 17 '11 at 19:48





$begingroup$
You aren't computing the parity correctly. The multiplication rule should be the same for all $S_n$. Also, $A_n$ embeds into $A_n+1$ for all $n$.
$endgroup$
– Qiaochu Yuan
Nov 17 '11 at 19:48













$begingroup$
@bwkaplan: Why did you change back the title? The question asks that you embed $S_n$ into $A_n+2$, not into $A_n+3$. Your entire question never mentions $A_n+3$.
$endgroup$
– Arturo Magidin
Nov 17 '11 at 20:04




$begingroup$
@bwkaplan: Why did you change back the title? The question asks that you embed $S_n$ into $A_n+2$, not into $A_n+3$. Your entire question never mentions $A_n+3$.
$endgroup$
– Arturo Magidin
Nov 17 '11 at 20:04












$begingroup$
@ArturoMagidin, I am more interested in the $A_2n+1$ groups, with the first one e.g. $A_n+1$ being a special case. See the last paragraph. Anyway, I'm going off to recalculate the parity again and will be back if I have any more questions. thanks again!
$endgroup$
– user4536
Nov 17 '11 at 20:18




$begingroup$
@ArturoMagidin, I am more interested in the $A_2n+1$ groups, with the first one e.g. $A_n+1$ being a special case. See the last paragraph. Anyway, I'm going off to recalculate the parity again and will be back if I have any more questions. thanks again!
$endgroup$
– user4536
Nov 17 '11 at 20:18












$begingroup$
@bwkaplan: I think only Qiaochu Yuan has read your (mind to be able to read your) actual question, but perhaps his answer was not direct enough. Yes, Sn is isomorphic to a subgroup of A(n+3). You answer the exercise to get Sn isomorphic to a subgroup of A(n+2), and then use QY's answer that A(n+2) is isomorphic to a subgroup of A(n+3).
$endgroup$
– Jack Schmidt
Nov 17 '11 at 20:22





$begingroup$
@bwkaplan: I think only Qiaochu Yuan has read your (mind to be able to read your) actual question, but perhaps his answer was not direct enough. Yes, Sn is isomorphic to a subgroup of A(n+3). You answer the exercise to get Sn isomorphic to a subgroup of A(n+2), and then use QY's answer that A(n+2) is isomorphic to a subgroup of A(n+3).
$endgroup$
– Jack Schmidt
Nov 17 '11 at 20:22













$begingroup$
@bwkaplan: Still, $2n+1neq n+3$...
$endgroup$
– Arturo Magidin
Nov 17 '11 at 20:30




$begingroup$
@bwkaplan: Still, $2n+1neq n+3$...
$endgroup$
– Arturo Magidin
Nov 17 '11 at 20:30










3 Answers
3






active

oldest

votes


















8












$begingroup$

I don't think you understand even and odd permutations. Odd * odd or even * even is even, odd * even or even * odd is odd. It doesn't matter what $n$ is.



You can embed $S_n$ in $A_n+2$ by the map $f$ where $f(sigma) = sigma$ if $sigma$ is even and $f(sigma) = sigma cdot (n+1,n+2)$ if $sigma$ is odd.






share|cite|improve this answer









$endgroup$












  • $begingroup$
    can you please explain why $f(sigma) = sigma cdot (n+1, n+2)$ is an even permutation?
    $endgroup$
    – ALannister
    Nov 7 '16 at 14:34










  • $begingroup$
    A permutation is even or odd depending on whether it is the product of an even or odd number of transpositions. So the product of an odd permutation and a transposition is even.
    $endgroup$
    – Robert Israel
    Nov 7 '16 at 16:51


















8












$begingroup$

You are not computing the parity correctly. In any symmetric group, the product of two even permutations is an even permutation (otherwise, $A_n$ would not be a subgroup!), the product of two odd permutations is an even permutation; and the product of two permutations of different parity is an odd permutation.



Instead, let's think about $S_3$, and how to embed it (or be unable to embed it) in $A_4$ and $A_5$.



$$S_3 = mathrmId, (1,2), (1,3), (2,3), (1,2,3), (1,3,2).$$
Now, $A_4$ doesn't have any subgroups of index $2$: they would be normal, and the only normal subgroups of $A_4$ are the trivial group, they whole group, and the subgroup $mathrmId, (1,2)(3,4), (1,3)(2,4), (1,4)(2,3)$ of index $3$. So $S_3$ cannot possibly embed into $A_4$.



What about $A_5$? There we have more leeway: there is no problem in mapping $(1,2,3)$ and $(1,3,2)$ to $A_5$: they have to map to $3$-cycles, so up to an automorphism of $A_5$ we may assume that we send $(1,2,3)$ to itself, and therefore $(1,3,2)$ to itself as well. We cannot just map $(1,2)$ to itself, though: $(1,2)notin A_5$. However, we can "fix" the parity by multiplying by a single that won't mess up the action on $1,2,3$ (so that we still get that the product of the image of $(1,2)$ and the image of $(1,3)$ is $(1,3,2)$, as needed, since $(1,2)(1,3)=(1,3,2)$). Just take $(1,2)(4,5)$! Similarly, try sending $(1,3)$ to $(1,3)(4,5)$, and $(2,3)$ to $(2,3)(4,5)$. Check that everything works out.



You may have to do the small $n$ cases separately, but after $n=3$, it should be simple to see how to get this done.



Added. Moreover, since $S_nhookrightarrow S_n+1$ (as the subgroup that fixes $n$), and this embedding necessarily respects parity, you have that $S_n$ embedds into $S_k$ for all $kgeq n$, and $A_n$ embeds into $A_k$ for all $kgeq n$. Added to the exercise, this shows that:



Proposition. Let $n$ be a positive integer. Then $S_n$ embeds into $A_k$ if and only if $n=1$ and $k$ is arbitrary; or $ngt 1$ and $kgeq n+2$.



For your title question, the answer is "yes" (if $ngt 1$, then $2n+1gt n+2$; if $n=1$, then $S_n$ is trivial and embeds into every $A_n$, in particular into $A_2$, which is also trivial).




More interesting is the question of when $S_n$ embeds into $A_k$ as a maximal subgroup. The pairs $(n,k)$ with this property were completely characterized by Bret Benesh in A classification of certain maximal subgroups of alternating groups, in Computational Group Theory and the Theory of Groups, Contemporary Mathematics vol. 470, American Mathematical Society, pages 21-26.






share|cite|improve this answer











$endgroup$








  • 1




    $begingroup$
    I don't think this proof is complete, when did you show $S_n$ does not embed in $A_n+1$?
    $endgroup$
    – Jorge Fernández Hidalgo
    Jun 2 '17 at 8:46










  • $begingroup$
    For a proof of the proposition, see mathoverflow.net/questions/65083
    $endgroup$
    – rabota
    Apr 1 at 8:07


















2












$begingroup$

That $S_n$ does not embed into $A_n+1$ for $nge 2$ is not proved in this thread (at this time), but a proof can be found at this MO post. Since this post has been used to close subsequent occurrences of this question as duplicate, I'm adding this cw answer for completeness.



For $n$ even it's clear since then $|S_n|$ does not divide $|A_n+1|$. In general, one proof essentially consists in showing that, for $nge 4$ the centralizer of any element of order $2$ in $A_n+1$ is smaller than that of a transposition in $S_n$, which requires a little argument. Alternatively one uses that the image would have index $(n+1)!/2<n+1$, which contradicts simplicity of $A_n+1$ when $nge 4$. For $n=3$, using that $mathrmHom(S_3,C_3)=1$ while $mathrmHom(A_4,C_3)neq1$, one sees that the image of such an embedding would be contained in the normal subgroup of index 3 of $A_4$, of order 4, to get a contradiction.






share|cite|improve this answer











$endgroup$













    Your Answer








    StackExchange.ready(function()
    var channelOptions =
    tags: "".split(" "),
    id: "69"
    ;
    initTagRenderer("".split(" "), "".split(" "), channelOptions);

    StackExchange.using("externalEditor", function()
    // Have to fire editor after snippets, if snippets enabled
    if (StackExchange.settings.snippets.snippetsEnabled)
    StackExchange.using("snippets", function()
    createEditor();
    );

    else
    createEditor();

    );

    function createEditor()
    StackExchange.prepareEditor(
    heartbeatType: 'answer',
    autoActivateHeartbeat: false,
    convertImagesToLinks: true,
    noModals: true,
    showLowRepImageUploadWarning: true,
    reputationToPostImages: 10,
    bindNavPrevention: true,
    postfix: "",
    imageUploader:
    brandingHtml: "Powered by u003ca class="icon-imgur-white" href="https://imgur.com/"u003eu003c/au003e",
    contentPolicyHtml: "User contributions licensed under u003ca href="https://creativecommons.org/licenses/by-sa/3.0/"u003ecc by-sa 3.0 with attribution requiredu003c/au003e u003ca href="https://stackoverflow.com/legal/content-policy"u003e(content policy)u003c/au003e",
    allowUrls: true
    ,
    noCode: true, onDemand: true,
    discardSelector: ".discard-answer"
    ,immediatelyShowMarkdownHelp:true
    );



    );













    draft saved

    draft discarded


















    StackExchange.ready(
    function ()
    StackExchange.openid.initPostLogin('.new-post-login', 'https%3a%2f%2fmath.stackexchange.com%2fquestions%2f83160%2fcan-the-symmetric-group-s-n-be-imbedded-as-a-subgroup-in-a-n1%23new-answer', 'question_page');

    );

    Post as a guest















    Required, but never shown
























    3 Answers
    3






    active

    oldest

    votes








    3 Answers
    3






    active

    oldest

    votes









    active

    oldest

    votes






    active

    oldest

    votes









    8












    $begingroup$

    I don't think you understand even and odd permutations. Odd * odd or even * even is even, odd * even or even * odd is odd. It doesn't matter what $n$ is.



    You can embed $S_n$ in $A_n+2$ by the map $f$ where $f(sigma) = sigma$ if $sigma$ is even and $f(sigma) = sigma cdot (n+1,n+2)$ if $sigma$ is odd.






    share|cite|improve this answer









    $endgroup$












    • $begingroup$
      can you please explain why $f(sigma) = sigma cdot (n+1, n+2)$ is an even permutation?
      $endgroup$
      – ALannister
      Nov 7 '16 at 14:34










    • $begingroup$
      A permutation is even or odd depending on whether it is the product of an even or odd number of transpositions. So the product of an odd permutation and a transposition is even.
      $endgroup$
      – Robert Israel
      Nov 7 '16 at 16:51















    8












    $begingroup$

    I don't think you understand even and odd permutations. Odd * odd or even * even is even, odd * even or even * odd is odd. It doesn't matter what $n$ is.



    You can embed $S_n$ in $A_n+2$ by the map $f$ where $f(sigma) = sigma$ if $sigma$ is even and $f(sigma) = sigma cdot (n+1,n+2)$ if $sigma$ is odd.






    share|cite|improve this answer









    $endgroup$












    • $begingroup$
      can you please explain why $f(sigma) = sigma cdot (n+1, n+2)$ is an even permutation?
      $endgroup$
      – ALannister
      Nov 7 '16 at 14:34










    • $begingroup$
      A permutation is even or odd depending on whether it is the product of an even or odd number of transpositions. So the product of an odd permutation and a transposition is even.
      $endgroup$
      – Robert Israel
      Nov 7 '16 at 16:51













    8












    8








    8





    $begingroup$

    I don't think you understand even and odd permutations. Odd * odd or even * even is even, odd * even or even * odd is odd. It doesn't matter what $n$ is.



    You can embed $S_n$ in $A_n+2$ by the map $f$ where $f(sigma) = sigma$ if $sigma$ is even and $f(sigma) = sigma cdot (n+1,n+2)$ if $sigma$ is odd.






    share|cite|improve this answer









    $endgroup$



    I don't think you understand even and odd permutations. Odd * odd or even * even is even, odd * even or even * odd is odd. It doesn't matter what $n$ is.



    You can embed $S_n$ in $A_n+2$ by the map $f$ where $f(sigma) = sigma$ if $sigma$ is even and $f(sigma) = sigma cdot (n+1,n+2)$ if $sigma$ is odd.







    share|cite|improve this answer












    share|cite|improve this answer



    share|cite|improve this answer










    answered Nov 17 '11 at 20:03









    Robert IsraelRobert Israel

    332k23222479




    332k23222479











    • $begingroup$
      can you please explain why $f(sigma) = sigma cdot (n+1, n+2)$ is an even permutation?
      $endgroup$
      – ALannister
      Nov 7 '16 at 14:34










    • $begingroup$
      A permutation is even or odd depending on whether it is the product of an even or odd number of transpositions. So the product of an odd permutation and a transposition is even.
      $endgroup$
      – Robert Israel
      Nov 7 '16 at 16:51
















    • $begingroup$
      can you please explain why $f(sigma) = sigma cdot (n+1, n+2)$ is an even permutation?
      $endgroup$
      – ALannister
      Nov 7 '16 at 14:34










    • $begingroup$
      A permutation is even or odd depending on whether it is the product of an even or odd number of transpositions. So the product of an odd permutation and a transposition is even.
      $endgroup$
      – Robert Israel
      Nov 7 '16 at 16:51















    $begingroup$
    can you please explain why $f(sigma) = sigma cdot (n+1, n+2)$ is an even permutation?
    $endgroup$
    – ALannister
    Nov 7 '16 at 14:34




    $begingroup$
    can you please explain why $f(sigma) = sigma cdot (n+1, n+2)$ is an even permutation?
    $endgroup$
    – ALannister
    Nov 7 '16 at 14:34












    $begingroup$
    A permutation is even or odd depending on whether it is the product of an even or odd number of transpositions. So the product of an odd permutation and a transposition is even.
    $endgroup$
    – Robert Israel
    Nov 7 '16 at 16:51




    $begingroup$
    A permutation is even or odd depending on whether it is the product of an even or odd number of transpositions. So the product of an odd permutation and a transposition is even.
    $endgroup$
    – Robert Israel
    Nov 7 '16 at 16:51











    8












    $begingroup$

    You are not computing the parity correctly. In any symmetric group, the product of two even permutations is an even permutation (otherwise, $A_n$ would not be a subgroup!), the product of two odd permutations is an even permutation; and the product of two permutations of different parity is an odd permutation.



    Instead, let's think about $S_3$, and how to embed it (or be unable to embed it) in $A_4$ and $A_5$.



    $$S_3 = mathrmId, (1,2), (1,3), (2,3), (1,2,3), (1,3,2).$$
    Now, $A_4$ doesn't have any subgroups of index $2$: they would be normal, and the only normal subgroups of $A_4$ are the trivial group, they whole group, and the subgroup $mathrmId, (1,2)(3,4), (1,3)(2,4), (1,4)(2,3)$ of index $3$. So $S_3$ cannot possibly embed into $A_4$.



    What about $A_5$? There we have more leeway: there is no problem in mapping $(1,2,3)$ and $(1,3,2)$ to $A_5$: they have to map to $3$-cycles, so up to an automorphism of $A_5$ we may assume that we send $(1,2,3)$ to itself, and therefore $(1,3,2)$ to itself as well. We cannot just map $(1,2)$ to itself, though: $(1,2)notin A_5$. However, we can "fix" the parity by multiplying by a single that won't mess up the action on $1,2,3$ (so that we still get that the product of the image of $(1,2)$ and the image of $(1,3)$ is $(1,3,2)$, as needed, since $(1,2)(1,3)=(1,3,2)$). Just take $(1,2)(4,5)$! Similarly, try sending $(1,3)$ to $(1,3)(4,5)$, and $(2,3)$ to $(2,3)(4,5)$. Check that everything works out.



    You may have to do the small $n$ cases separately, but after $n=3$, it should be simple to see how to get this done.



    Added. Moreover, since $S_nhookrightarrow S_n+1$ (as the subgroup that fixes $n$), and this embedding necessarily respects parity, you have that $S_n$ embedds into $S_k$ for all $kgeq n$, and $A_n$ embeds into $A_k$ for all $kgeq n$. Added to the exercise, this shows that:



    Proposition. Let $n$ be a positive integer. Then $S_n$ embeds into $A_k$ if and only if $n=1$ and $k$ is arbitrary; or $ngt 1$ and $kgeq n+2$.



    For your title question, the answer is "yes" (if $ngt 1$, then $2n+1gt n+2$; if $n=1$, then $S_n$ is trivial and embeds into every $A_n$, in particular into $A_2$, which is also trivial).




    More interesting is the question of when $S_n$ embeds into $A_k$ as a maximal subgroup. The pairs $(n,k)$ with this property were completely characterized by Bret Benesh in A classification of certain maximal subgroups of alternating groups, in Computational Group Theory and the Theory of Groups, Contemporary Mathematics vol. 470, American Mathematical Society, pages 21-26.






    share|cite|improve this answer











    $endgroup$








    • 1




      $begingroup$
      I don't think this proof is complete, when did you show $S_n$ does not embed in $A_n+1$?
      $endgroup$
      – Jorge Fernández Hidalgo
      Jun 2 '17 at 8:46










    • $begingroup$
      For a proof of the proposition, see mathoverflow.net/questions/65083
      $endgroup$
      – rabota
      Apr 1 at 8:07















    8












    $begingroup$

    You are not computing the parity correctly. In any symmetric group, the product of two even permutations is an even permutation (otherwise, $A_n$ would not be a subgroup!), the product of two odd permutations is an even permutation; and the product of two permutations of different parity is an odd permutation.



    Instead, let's think about $S_3$, and how to embed it (or be unable to embed it) in $A_4$ and $A_5$.



    $$S_3 = mathrmId, (1,2), (1,3), (2,3), (1,2,3), (1,3,2).$$
    Now, $A_4$ doesn't have any subgroups of index $2$: they would be normal, and the only normal subgroups of $A_4$ are the trivial group, they whole group, and the subgroup $mathrmId, (1,2)(3,4), (1,3)(2,4), (1,4)(2,3)$ of index $3$. So $S_3$ cannot possibly embed into $A_4$.



    What about $A_5$? There we have more leeway: there is no problem in mapping $(1,2,3)$ and $(1,3,2)$ to $A_5$: they have to map to $3$-cycles, so up to an automorphism of $A_5$ we may assume that we send $(1,2,3)$ to itself, and therefore $(1,3,2)$ to itself as well. We cannot just map $(1,2)$ to itself, though: $(1,2)notin A_5$. However, we can "fix" the parity by multiplying by a single that won't mess up the action on $1,2,3$ (so that we still get that the product of the image of $(1,2)$ and the image of $(1,3)$ is $(1,3,2)$, as needed, since $(1,2)(1,3)=(1,3,2)$). Just take $(1,2)(4,5)$! Similarly, try sending $(1,3)$ to $(1,3)(4,5)$, and $(2,3)$ to $(2,3)(4,5)$. Check that everything works out.



    You may have to do the small $n$ cases separately, but after $n=3$, it should be simple to see how to get this done.



    Added. Moreover, since $S_nhookrightarrow S_n+1$ (as the subgroup that fixes $n$), and this embedding necessarily respects parity, you have that $S_n$ embedds into $S_k$ for all $kgeq n$, and $A_n$ embeds into $A_k$ for all $kgeq n$. Added to the exercise, this shows that:



    Proposition. Let $n$ be a positive integer. Then $S_n$ embeds into $A_k$ if and only if $n=1$ and $k$ is arbitrary; or $ngt 1$ and $kgeq n+2$.



    For your title question, the answer is "yes" (if $ngt 1$, then $2n+1gt n+2$; if $n=1$, then $S_n$ is trivial and embeds into every $A_n$, in particular into $A_2$, which is also trivial).




    More interesting is the question of when $S_n$ embeds into $A_k$ as a maximal subgroup. The pairs $(n,k)$ with this property were completely characterized by Bret Benesh in A classification of certain maximal subgroups of alternating groups, in Computational Group Theory and the Theory of Groups, Contemporary Mathematics vol. 470, American Mathematical Society, pages 21-26.






    share|cite|improve this answer











    $endgroup$








    • 1




      $begingroup$
      I don't think this proof is complete, when did you show $S_n$ does not embed in $A_n+1$?
      $endgroup$
      – Jorge Fernández Hidalgo
      Jun 2 '17 at 8:46










    • $begingroup$
      For a proof of the proposition, see mathoverflow.net/questions/65083
      $endgroup$
      – rabota
      Apr 1 at 8:07













    8












    8








    8





    $begingroup$

    You are not computing the parity correctly. In any symmetric group, the product of two even permutations is an even permutation (otherwise, $A_n$ would not be a subgroup!), the product of two odd permutations is an even permutation; and the product of two permutations of different parity is an odd permutation.



    Instead, let's think about $S_3$, and how to embed it (or be unable to embed it) in $A_4$ and $A_5$.



    $$S_3 = mathrmId, (1,2), (1,3), (2,3), (1,2,3), (1,3,2).$$
    Now, $A_4$ doesn't have any subgroups of index $2$: they would be normal, and the only normal subgroups of $A_4$ are the trivial group, they whole group, and the subgroup $mathrmId, (1,2)(3,4), (1,3)(2,4), (1,4)(2,3)$ of index $3$. So $S_3$ cannot possibly embed into $A_4$.



    What about $A_5$? There we have more leeway: there is no problem in mapping $(1,2,3)$ and $(1,3,2)$ to $A_5$: they have to map to $3$-cycles, so up to an automorphism of $A_5$ we may assume that we send $(1,2,3)$ to itself, and therefore $(1,3,2)$ to itself as well. We cannot just map $(1,2)$ to itself, though: $(1,2)notin A_5$. However, we can "fix" the parity by multiplying by a single that won't mess up the action on $1,2,3$ (so that we still get that the product of the image of $(1,2)$ and the image of $(1,3)$ is $(1,3,2)$, as needed, since $(1,2)(1,3)=(1,3,2)$). Just take $(1,2)(4,5)$! Similarly, try sending $(1,3)$ to $(1,3)(4,5)$, and $(2,3)$ to $(2,3)(4,5)$. Check that everything works out.



    You may have to do the small $n$ cases separately, but after $n=3$, it should be simple to see how to get this done.



    Added. Moreover, since $S_nhookrightarrow S_n+1$ (as the subgroup that fixes $n$), and this embedding necessarily respects parity, you have that $S_n$ embedds into $S_k$ for all $kgeq n$, and $A_n$ embeds into $A_k$ for all $kgeq n$. Added to the exercise, this shows that:



    Proposition. Let $n$ be a positive integer. Then $S_n$ embeds into $A_k$ if and only if $n=1$ and $k$ is arbitrary; or $ngt 1$ and $kgeq n+2$.



    For your title question, the answer is "yes" (if $ngt 1$, then $2n+1gt n+2$; if $n=1$, then $S_n$ is trivial and embeds into every $A_n$, in particular into $A_2$, which is also trivial).




    More interesting is the question of when $S_n$ embeds into $A_k$ as a maximal subgroup. The pairs $(n,k)$ with this property were completely characterized by Bret Benesh in A classification of certain maximal subgroups of alternating groups, in Computational Group Theory and the Theory of Groups, Contemporary Mathematics vol. 470, American Mathematical Society, pages 21-26.






    share|cite|improve this answer











    $endgroup$



    You are not computing the parity correctly. In any symmetric group, the product of two even permutations is an even permutation (otherwise, $A_n$ would not be a subgroup!), the product of two odd permutations is an even permutation; and the product of two permutations of different parity is an odd permutation.



    Instead, let's think about $S_3$, and how to embed it (or be unable to embed it) in $A_4$ and $A_5$.



    $$S_3 = mathrmId, (1,2), (1,3), (2,3), (1,2,3), (1,3,2).$$
    Now, $A_4$ doesn't have any subgroups of index $2$: they would be normal, and the only normal subgroups of $A_4$ are the trivial group, they whole group, and the subgroup $mathrmId, (1,2)(3,4), (1,3)(2,4), (1,4)(2,3)$ of index $3$. So $S_3$ cannot possibly embed into $A_4$.



    What about $A_5$? There we have more leeway: there is no problem in mapping $(1,2,3)$ and $(1,3,2)$ to $A_5$: they have to map to $3$-cycles, so up to an automorphism of $A_5$ we may assume that we send $(1,2,3)$ to itself, and therefore $(1,3,2)$ to itself as well. We cannot just map $(1,2)$ to itself, though: $(1,2)notin A_5$. However, we can "fix" the parity by multiplying by a single that won't mess up the action on $1,2,3$ (so that we still get that the product of the image of $(1,2)$ and the image of $(1,3)$ is $(1,3,2)$, as needed, since $(1,2)(1,3)=(1,3,2)$). Just take $(1,2)(4,5)$! Similarly, try sending $(1,3)$ to $(1,3)(4,5)$, and $(2,3)$ to $(2,3)(4,5)$. Check that everything works out.



    You may have to do the small $n$ cases separately, but after $n=3$, it should be simple to see how to get this done.



    Added. Moreover, since $S_nhookrightarrow S_n+1$ (as the subgroup that fixes $n$), and this embedding necessarily respects parity, you have that $S_n$ embedds into $S_k$ for all $kgeq n$, and $A_n$ embeds into $A_k$ for all $kgeq n$. Added to the exercise, this shows that:



    Proposition. Let $n$ be a positive integer. Then $S_n$ embeds into $A_k$ if and only if $n=1$ and $k$ is arbitrary; or $ngt 1$ and $kgeq n+2$.



    For your title question, the answer is "yes" (if $ngt 1$, then $2n+1gt n+2$; if $n=1$, then $S_n$ is trivial and embeds into every $A_n$, in particular into $A_2$, which is also trivial).




    More interesting is the question of when $S_n$ embeds into $A_k$ as a maximal subgroup. The pairs $(n,k)$ with this property were completely characterized by Bret Benesh in A classification of certain maximal subgroups of alternating groups, in Computational Group Theory and the Theory of Groups, Contemporary Mathematics vol. 470, American Mathematical Society, pages 21-26.







    share|cite|improve this answer














    share|cite|improve this answer



    share|cite|improve this answer








    edited Nov 17 '11 at 22:19

























    answered Nov 17 '11 at 20:03









    Arturo MagidinArturo Magidin

    267k34591922




    267k34591922







    • 1




      $begingroup$
      I don't think this proof is complete, when did you show $S_n$ does not embed in $A_n+1$?
      $endgroup$
      – Jorge Fernández Hidalgo
      Jun 2 '17 at 8:46










    • $begingroup$
      For a proof of the proposition, see mathoverflow.net/questions/65083
      $endgroup$
      – rabota
      Apr 1 at 8:07












    • 1




      $begingroup$
      I don't think this proof is complete, when did you show $S_n$ does not embed in $A_n+1$?
      $endgroup$
      – Jorge Fernández Hidalgo
      Jun 2 '17 at 8:46










    • $begingroup$
      For a proof of the proposition, see mathoverflow.net/questions/65083
      $endgroup$
      – rabota
      Apr 1 at 8:07







    1




    1




    $begingroup$
    I don't think this proof is complete, when did you show $S_n$ does not embed in $A_n+1$?
    $endgroup$
    – Jorge Fernández Hidalgo
    Jun 2 '17 at 8:46




    $begingroup$
    I don't think this proof is complete, when did you show $S_n$ does not embed in $A_n+1$?
    $endgroup$
    – Jorge Fernández Hidalgo
    Jun 2 '17 at 8:46












    $begingroup$
    For a proof of the proposition, see mathoverflow.net/questions/65083
    $endgroup$
    – rabota
    Apr 1 at 8:07




    $begingroup$
    For a proof of the proposition, see mathoverflow.net/questions/65083
    $endgroup$
    – rabota
    Apr 1 at 8:07











    2












    $begingroup$

    That $S_n$ does not embed into $A_n+1$ for $nge 2$ is not proved in this thread (at this time), but a proof can be found at this MO post. Since this post has been used to close subsequent occurrences of this question as duplicate, I'm adding this cw answer for completeness.



    For $n$ even it's clear since then $|S_n|$ does not divide $|A_n+1|$. In general, one proof essentially consists in showing that, for $nge 4$ the centralizer of any element of order $2$ in $A_n+1$ is smaller than that of a transposition in $S_n$, which requires a little argument. Alternatively one uses that the image would have index $(n+1)!/2<n+1$, which contradicts simplicity of $A_n+1$ when $nge 4$. For $n=3$, using that $mathrmHom(S_3,C_3)=1$ while $mathrmHom(A_4,C_3)neq1$, one sees that the image of such an embedding would be contained in the normal subgroup of index 3 of $A_4$, of order 4, to get a contradiction.






    share|cite|improve this answer











    $endgroup$

















      2












      $begingroup$

      That $S_n$ does not embed into $A_n+1$ for $nge 2$ is not proved in this thread (at this time), but a proof can be found at this MO post. Since this post has been used to close subsequent occurrences of this question as duplicate, I'm adding this cw answer for completeness.



      For $n$ even it's clear since then $|S_n|$ does not divide $|A_n+1|$. In general, one proof essentially consists in showing that, for $nge 4$ the centralizer of any element of order $2$ in $A_n+1$ is smaller than that of a transposition in $S_n$, which requires a little argument. Alternatively one uses that the image would have index $(n+1)!/2<n+1$, which contradicts simplicity of $A_n+1$ when $nge 4$. For $n=3$, using that $mathrmHom(S_3,C_3)=1$ while $mathrmHom(A_4,C_3)neq1$, one sees that the image of such an embedding would be contained in the normal subgroup of index 3 of $A_4$, of order 4, to get a contradiction.






      share|cite|improve this answer











      $endgroup$















        2












        2








        2





        $begingroup$

        That $S_n$ does not embed into $A_n+1$ for $nge 2$ is not proved in this thread (at this time), but a proof can be found at this MO post. Since this post has been used to close subsequent occurrences of this question as duplicate, I'm adding this cw answer for completeness.



        For $n$ even it's clear since then $|S_n|$ does not divide $|A_n+1|$. In general, one proof essentially consists in showing that, for $nge 4$ the centralizer of any element of order $2$ in $A_n+1$ is smaller than that of a transposition in $S_n$, which requires a little argument. Alternatively one uses that the image would have index $(n+1)!/2<n+1$, which contradicts simplicity of $A_n+1$ when $nge 4$. For $n=3$, using that $mathrmHom(S_3,C_3)=1$ while $mathrmHom(A_4,C_3)neq1$, one sees that the image of such an embedding would be contained in the normal subgroup of index 3 of $A_4$, of order 4, to get a contradiction.






        share|cite|improve this answer











        $endgroup$



        That $S_n$ does not embed into $A_n+1$ for $nge 2$ is not proved in this thread (at this time), but a proof can be found at this MO post. Since this post has been used to close subsequent occurrences of this question as duplicate, I'm adding this cw answer for completeness.



        For $n$ even it's clear since then $|S_n|$ does not divide $|A_n+1|$. In general, one proof essentially consists in showing that, for $nge 4$ the centralizer of any element of order $2$ in $A_n+1$ is smaller than that of a transposition in $S_n$, which requires a little argument. Alternatively one uses that the image would have index $(n+1)!/2<n+1$, which contradicts simplicity of $A_n+1$ when $nge 4$. For $n=3$, using that $mathrmHom(S_3,C_3)=1$ while $mathrmHom(A_4,C_3)neq1$, one sees that the image of such an embedding would be contained in the normal subgroup of index 3 of $A_4$, of order 4, to get a contradiction.







        share|cite|improve this answer














        share|cite|improve this answer



        share|cite|improve this answer








        answered Apr 1 at 10:11


























        community wiki





        YCor




























            draft saved

            draft discarded
















































            Thanks for contributing an answer to Mathematics Stack Exchange!


            • Please be sure to answer the question. Provide details and share your research!

            But avoid


            • Asking for help, clarification, or responding to other answers.

            • Making statements based on opinion; back them up with references or personal experience.

            Use MathJax to format equations. MathJax reference.


            To learn more, see our tips on writing great answers.




            draft saved


            draft discarded














            StackExchange.ready(
            function ()
            StackExchange.openid.initPostLogin('.new-post-login', 'https%3a%2f%2fmath.stackexchange.com%2fquestions%2f83160%2fcan-the-symmetric-group-s-n-be-imbedded-as-a-subgroup-in-a-n1%23new-answer', 'question_page');

            );

            Post as a guest















            Required, but never shown





















































            Required, but never shown














            Required, but never shown












            Required, but never shown







            Required, but never shown

































            Required, but never shown














            Required, but never shown












            Required, but never shown







            Required, but never shown







            Popular posts from this blog

            Triangular numbers and gcdProving sum of a set is $0 pmod n$ if $n$ is odd, or $fracn2 pmod n$ if $n$ is even?Is greatest common divisor of two numbers really their smallest linear combination?GCD, LCM RelationshipProve a set of nonnegative integers with greatest common divisor 1 and closed under addition has all but finite many nonnegative integers.all pairs of a and b in an equation containing gcdTriangular Numbers Modulo $k$ - Hit All Values?Understanding the Existence and Uniqueness of the GCDGCD and LCM with logical symbolsThe greatest common divisor of two positive integers less than 100 is equal to 3. Their least common multiple is twelve times one of the integers.Suppose that for all integers $x$, $x|a$ and $x|b$ if and only if $x|c$. Then $c = gcd(a,b)$Which is the gcd of 2 numbers which are multiplied and the result is 600000?

            Ingelân Ynhâld Etymology | Geografy | Skiednis | Polityk en bestjoer | Ekonomy | Demografy | Kultuer | Klimaat | Sjoch ek | Keppelings om utens | Boarnen, noaten en referinsjes Navigaasjemenuwww.gov.ukOffisjele webside fan it regear fan it Feriene KeninkrykOffisjele webside fan it Britske FerkearsburoNederlânsktalige ynformaasje fan it Britske FerkearsburoOffisjele webside fan English Heritage, de organisaasje dy't him ynset foar it behâld fan it Ingelske kultuergoedYnwennertallen fan alle Britske stêden út 'e folkstelling fan 2011Notes en References, op dizze sideEngland

            Հադիս Բովանդակություն Անվանում և նշանակություն | Դասակարգում | Աղբյուրներ | Նավարկման ցանկ